(a) {(1)¹⁰⁰ + (−1)¹⁰¹+(2500)⁰} x2³​

Answers

Answer 1

Answer:

8

Step-by-step explanation:

You should have this in mind , one raise to the power of any number is one and also any number with the power zero is 1.

{(1)¹⁰⁰ + (−1)¹⁰¹+(2500)⁰} x2³

= 1 - 1 + 1 × 8

= 1 - 1 + 8

= 1 +7

= 8


Related Questions

Describe the change in the graph of the parabola f(x) when it transforms into g(x) =
The parabola g(x) will open in the opposite direction of f(x), and the parabola will be narrower than f(x).
The parabola g(x) will open in the same direction of f(x), and the parabola will be narrower than f(x).
The parabola g(x) will open in the opposite direction of f(x), and the parabola will be wider than f(x).
The parabola g(x) will open in the same direction of f(x), and the parabola will be wider than f(x).

Answers

Answer:

  (d)  The parabola g(x) will open in the same direction of f(x), and the parabola will be wider than f(x).

Step-by-step explanation:

We assume you intend ...

  f(x) = equation of a parabola

  g(x) = 2/3·f(x)

Multiplying a function by a factor of 2/3 will cause it to be compressed vertically to 2/3 of its original height. When the function is a parabola, this has the effect of making it appear wider than before the compression.

__

The compression factor is positive, so points on the graph remain on the same side of the x-axis. The direction in which the graph opens is not changed.

The attachment shows parabolas that open upward and downward, along with the transformed version.

Answer:

D.)  The parabola g(x) will open in the same direction of f(x), and the parabola will be wider than f(x).

Step-by-step explanation:

I got it right on the test :)

stay hydrated.

Where does billy bob johns

Answers

Answer:

in ur moms house

Step-by-step explanation:

Answer:

he doesn't

Step-by-step explanation:

Find the area of a 150° sector of a circle whose radius is 3.

Answers

Answer: [tex]\frac{15\pi}{4}[/tex]

Step-by-step explanation:

[tex]A=\pi(3^{2})\left(\frac{150}{360} \right)=\boxed{\frac{15\pi}{4}}[/tex]

statement that is assumed to be true without proof is . A statement that has been shown to be true by vigorous application of logic is . is a statement that is believed to be true but hasn't been proven.

Answers

An axiom is a statement that is assumed to be true without proof.

What are the types of statements?

In logic, a statement could be shown to be valid or true by the use of premises that lead to a sound conclusion.

The following terms can be used to describe the statements;

Axiom -  a statement that is assumed to be true without proofTheorem - a statement that has been shown to be true by vigorous application of logicHypothesis -  a statement that is believed to be true but hasn't been proven.

Learn more about mathematical statements:https://brainly.com/question/17029275?

#SPJ1

Answer:

The answer is axiom, theorem, and then conjecture

Step-by-step explanation:

3xy - ² + 4x for x = -2, y = - and = 6?

Answers

Answer:

See below

Step-by-step explanation:

Not sure what you are asking....do you want to know what y is when x = -2   and the equation = 6 ?

3 xy^2 + 4x = 6         or is that    3 xy^(-2) + 4x = 6

Could use some serious editing of your question if you are able ....

help pls i jusiot jiojfgnio43nfginfmowenfviowngognior3

Answers

Answer:

i dont know

Step-by-step explanation:

sorry please be a bit more clear on what help you would like

Which expression is equivalent to One-fifth (150 x minus 80 y + 50 minus 50 x minus 25 y + 20)?

20 x minus 21 y + 14

20 x + 11 y + 14

20 x minus 11 y + 6

20 x + 21 y + 6

Answers

Answer:

150 x - 80 y + 50 - 50 x - 25 y + 20

Arranging like terms

150 x - 50 x -25 y - 80y + 50 + 20

100x - 105 y + 70

5 ( 20x - 21y + 14)

Answer:

20x - 21y + 14

Explanation:

[tex]\rightarrow \sf \dfrac{1}{5} (150x - 80y + 50 - 50x - 25y + 20)[/tex]

collect like terms

[tex]\rightarrow \sf \dfrac{1}{5} (150x-50x - 80y -25y + 50 + 20)[/tex]

add/subtract like terms

[tex]\rightarrow \sf \dfrac{1}{5} (100x - 105y+70)[/tex]

distribute inside parenthesis

[tex]\rightarrow \sf \dfrac{1}{5} (100x) + \dfrac{1}{5}( - 105y) + \dfrac{1}{5}(70)[/tex]

simplify the following

[tex]\rightarrow \sf 20x -21y + 14[/tex]

how do you simplify

−(12st3+34s3t+t2−1)

Answers

Answer:

-34s^3t - 12st^3 - t^2 + 1

Step-by-step explanation:

hope this helps

What is the value of x?
E
(2x)°

D

Answers

Answer:

30°

Step-by-step explanation:

Since ED = FD, the ∠DEF = ∠EFD
Since ∠DEF = 2x, ∠EFD is also equal to 2x
The sum of the 3 angles of a triangle is 180°

So x + 2x + 2x = 180
6x = 180
x = 30

A carpenter wants to cut an 84 inches long board into two-pieces such that one piece is twice as long as the other piece. Set up an equation and find the length of each piece.

Answers

It’s not an equation but the shorter piece would be 28 inches and the longer piece would be 56 inches

Answer:

small piece = 28 inches

large piece = 56 inches

Step-by-step explanation:

The carpenter has one piece of wood 84 inches long. He wants to cut that piece of wood into two pieces. One piece will be 2 times as long as the other. Let's let x = the small piece of wood

2x + x = 84

In this equation, the longer piece is 2 times the shorter piece and the shorter piece is added, which equals 84 inches. Lets solve:

2x + x = 84

3x = 84

3x/3 = 84/3

x = 28

This means that the shorter piece is 28 inches. To find the longer piece, multiply 28 by 2 because the longer piece is twice the shorter piece. Therefore the longer piece is

28 × 2 = 56

Finally, we figured out that the short piece is 28 inches and the longer piece is 56 inches

We can double check this by doing 56 + 28 and making sure it equals 84

PLEASE HELPP!!!!!
Question 9 of 13
Which of the following scatterplots represents the data shown below?
(1,31), (2,8), (3, 38), (4, 14), (5, 22), (6, 31), (7, 27), (8, 47),
(9,34), (10,3), (11, 33), (12, 35)

Answers

The attached graph below can represent the scatter plot of the points. the correct option is B.

How to find the function which was used to make graph?

A graph contains data of which input maps to which output.

Analysis of this leads to the relations which were used to make it.

Given:

(1,31), (2,8), (3, 38), (4, 14), (5, 22), (6, 31), (7, 27), (8, 47), (9,34), (10,3), (11, 33), (12, 35)

The above points would be plotted using the following scale such as:

x: axis -> 1 interval = 10 units

y: axis -> 1 interval = 10 units

The attached graph below can represent the scatter plot of the points. the correct option is B.

Read more about scatter plots at:

brainly.com/question/6592115

#SPJ1

which of the following expresses the coordinates of the foci of the conic section shown below? (x-2)^2/4+(y+5)^2/9

Answers

Step-by-step explanation:

[tex] \frac{(x - 2) {}^{2} }{4} + \frac{(y + 5) {}^{2} }{9} = 1[/tex]

This is the equation of the ellipse. Since the denominator is greater for the y values, we have a vertical ellipse. Remember a>b, so a

The formula for the foci of the vertical ellipse is

(h,k+c) and (h,k-c).

where c is

Our center (h,k) is (2, -5)

[tex] {c}^{2} = {a}^{2} - {b}^{2} [/tex]

Here a^2 is 9, b^2 is 4.

[tex] {c}^{2} = 9 - 4[/tex]

[tex] {c}^{2} = 5[/tex]

[tex]c = \sqrt{5} [/tex]

So our foci is

[tex](2, - 5 + \sqrt{5} )[/tex]

and

[tex](2, - 5 - \sqrt{5} )[/tex]

Find the value of: (4/5)^1 A. 4/5 B. 1 C. 0 D. 16/25​

Answers

Answer:

A. 4/5

Step-by-step explanation:

Answer:

4/5  (option a)

Step-by-step explanation:

by following the exponent rule of [tex]a^1 = a[/tex], we know that

[tex](\frac{4}{5} )^1[/tex] = [tex]\frac{4}{5}[/tex]

So, the value of

[tex]\frac{4}{5}^{1}[/tex]  is  4/5

A comedy club's nightly revenue from the sale of x tickets is given by R = 15x and its
nightly costs are given by C= 1.25x + 495. How many tickets need to be sold each night
to break even? In other words, when will revenue equal costs?

Answers

Considering the given equations for the revenue and the cost, it is found that 36 tickets have to be sold each night to break even.

What is the break-even point?

The break-even point is when the revenue and the costs are equal.

In this problem, the equations are given as follows:

R(x) = 15x.C(x) = 1.25x + 495.

At the break-even point:

R(x) = C(x)

15x = 1.25x + 495

13.75x = 495

x = 495/13.75

x = 36.

36 tickets have to be sold each night to break even.

More can be learned about break-even point at https://brainly.com/question/9212451

#SPJ1

Simplify
[tex]\left(\frac{123}{321}\right)\left(\frac{456}{654}\right)\left(\frac{789}{987}\right) \left(\frac{123}{321}\right)^{-1}\left(\frac{456}{654}\right)^{-1}\left(\frac{789}{987}\right)^{-1}[/tex]

Answers

Answer:

1

Step-by-step explanation:

using the rule of exponents

[tex](\frac{a}{b}) ^{-1}[/tex] = [tex]\frac{b}{a}[/tex] , then

[tex]\frac{123}{321}[/tex] × [tex]\frac{456}{654}[/tex] × [tex]\frac{789}{987}[/tex] × [tex]\frac{321}{123}[/tex] × [tex]\frac{654}{456}[/tex] × [tex]\frac{987}{789}[/tex]

[ [tex]\frac{123}{321}[/tex] × [tex]\frac{321}{123}[/tex] = 1 , [tex]\frac{456}{654}[/tex] × [tex]\frac{654}{456}[/tex] = 1 , [tex]\frac{789}{987}[/tex] × [tex]\frac{987}{789}[/tex] = 1 ]

= 1 × 1 × 1

= 1

If y=4 when x=12, find y when x=-24

Answers

The value of y is -8 if the x  -24 and y varies directly with x, and If y = 4 when x = 12.

What is a proportional relationship?

It is defined as the relationship between two variables when the first variable increases, the second variable also increases according to the constant factor.

The question is incomplete.

The complete question is:

If y varies directly with x, and If y = 4 when x = 12, how do you find y when x = -24?

y ∝ x  (given)

y = kx

k is the constant of proportionality.

4 = 12k  (y = 4, and x = 12)

k = 1/3

y = x/3

Plug x = -24

y = -24/3

y = -8

Thus, the value of y is -8 if the x  -24 and y varies directly with x, and If y = 4 when x = 12.

Learn more about the proportional here:

brainly.com/question/14263719

#SPJ1

Is AABCDEF? If so, identify the similarity postulate or theorem that
applies.
A
B
16 105⁰
36
с
D
4
LLI
E
9
-105⁰
F
OA. Similar - AA
OB. Similar - SSS
OC. Similar - SAS
OD. Cannot be determined
4

Answers

Answer:

C

Step-by-step explanation:

C is the correct answer as both triangles have a similar angle of 105⁰ and both side lengths of triangle ABC are being divided by 4 to have the same two side lengths as triangle DEF

The triangles ΔABC and ΔDEF are similar triangles by SAS theorem

What are similar triangles?

If two triangles' corresponding angles are congruent and their corresponding sides are proportional, they are said to be similar triangles. In other words, similar triangles have the same shape but may or may not be the same size. The triangles are congruent if their corresponding sides are also of identical length.

Corresponding sides of similar triangles are in the same ratio. The ratio of area of similar triangles is the same as the ratio of the square of any pair of their corresponding sides

Given data ,

Let the first triangle be ΔABC

The measure of side AB = 16

The measure of side AC = 36

And , the measure of ∠ABC = 105°

Let the first triangle be ΔDEF

The measure of side DE = 16

The measure of side DF = 36

And , the measure of ∠DEF = 105°

Now , the ratio of sides of the triangles is given by

AB / DE = AC / DF

4/16 = 9/36

1/4 = 1/4

So , corresponding sides of similar triangles are in the same ratio

And , the measure of angles = 105°

Therefore , by SAS , Two sides are equal and the angle between the two sides is equal (SAS: side, angle, side)

Hence , they are similar triangles

To learn more about similar triangles click :

https://brainly.com/question/29378183

#SPJ5

indicate whether the statement is true of false.
please provide a explanation


A linear system with three variables and three equations has a unique solution.

Answers

The statement is false, as the system can have no solutions or infinite solutions.

Is the statement true or false?

The statement says that a system of linear equations with 3 variables and 3 equations has one solution.

If the variables are x, y, and z, then the system can be written as:

[tex]a_1*x + b_1*y + c_1*z = d_1\\\\a_2*x + b_2*y + c_2*z = d_2\\\\a_3*x + b_3*y + c_3*z = d_3[/tex]

Now, the statement is clearly false. Suppose that we have:

[tex]a_1 = a_2 = a_3\\b_1 = b_2 = b_3\\c_1 = c_2 = c_3\\\\d_1 \neq d_2 \neq d_3[/tex]

Then we have 3 parallel equations. Parallel equations never do intercept, then this system has no solutions.

Then there are systems of 3 variables with 3 equations where there are no solutions, so the statement is false.

If you want to learn more about systems of equations:

https://brainly.com/question/13729904

#SPJ1

The equation for the pH of a substance is pH = -log[H*], where H+ is the concentration of hydrogen ions. A basic
solution has a pH of 11.2. An acidic solution has a pH of 2.4. What is the approximate difference in the concentratic
of hydrogen ions between the two solutions?
1.6x10-9
O4.0x10-3
O 6.7x10-1
O 1.6x10¹11

Answers

The approximate difference in the concentration of hydrogen ions between the two solutions is 4.0×10⁻³

What is pH value?

The pH of a solution is a measurement of the concentration of hydrogen ions in the solution, as well as its acidity or alkalinity. Normally, the pH scale ranges from 0 to 14.

pH value for basic solution = 11.2

pH value for acidic solution = 2.4

Concentration for a basic solution:

[tex]\rm = 10^{-11.2}[/tex]

[tex]= 6.309\times10^{-12}[/tex]

Concentration for an acidic solution:

[tex]\rm = 10^{-2.4}[/tex]

[tex]= 3.981\times10^{-3}[/tex]

Difference between the concentration:

[tex]= 3.981\times10^{-3}- 6.309\times10^{-12}[/tex]

[tex]= 3.98\times10^{-3}[/tex]

[tex]= 4\times10^{-3}[/tex]

Thus, the approximate difference in the concentration of hydrogen ions between the two solutions is 4.0×10⁻³

Learn more about the pH value here:

brainly.com/question/1503669

#SPJ1

Another day, another math problem 3

Answers

Answer:

[tex]2\sqrt{x+1}-3\\domain:[-1, \infty)[/tex]

Step-by-step explanation:

[tex]g(h(x)) = 2(\sqrt{x+1})-3\\g(h(x)) = 2\sqrt{x+1} - 3\\[/tex]

the function is only defined when (x+1) >= 0 (since square root) so the domain is when x >= -1

Select The correct answer.
Segment AB Is tangent to the circle at point B. Which equation describes the relationship between the tangent and secant line segments?
B
D
OA
(AD)² = (AC) (AB)
OB. (AB)2 = (AC)(CD)
OC. (AB)2 = (AC)(AD)
OD. AB=(AC + AD)

Answers

According to the tangent-secant theorem, the relationship of the tangent and secant in the given circle is: C. AB² = (AC)(AD).

What is the Tangent-Secant Theorem?

The secant tangent theorem defines the relationship between the lengths of the secant and the tangent line segments when they meet outside a circle.

Based on the tangent-secant theorem, using the image given, the equation that describes the relationship of the tangent and secant in the given circle is:

C. AB² = (AC)(AD)

Learn more about the tangent-secant theorem on:

https://brainly.com/question/4494553

#SPJ1

Answer: C

Step-by-step explanation:

got it right on my test!!

This is from Khan academy I have to attach a PNG if you can help me solve it! Thank you!

Answers

Step-by-step explanation:

[tex]29 {}^{ \frac{x}{2} } [/tex]

[tex]((29) {}^{ \frac{1}{2} } ) {}^{x} [/tex]

The answer is A

Word Problem 3 Priscilla works 43 hours a week for eight weeks. If she makes $14.73 per hour, how much money did she make in the eight weeks? Operation(s): Math sentence(s): Solution: Word Problem 3 Priscilla works 43 hours a week for eight weeks . If she makes $ 14.73 per hour , how much money did she make in the eight weeks ? Operation ( s ) : Math sentence ( s ) : Solution :​

Answers

Answer:

$5067,12

Step-by-step explanation:

14,73*43*8

The required money Priscilla will earn in 8 weeks is $5067.12.

As mentioned in the question, Priscilla works 43 hours a week for eight weeks. If she makes $14.73 per hour,

What is simplification?

The process in mathematics to operate and interpret the function to make the function or expression simple or more understandable is called simplifying and the process is called simplification.

Here,
Let the amount earned be x,
According to the condition,
x = 8 × 43 × 14.73
x =  $5067.12.

Thus, the required money Priscilla will earn in 8 weeks is $5067.12.

Learn more about simplification here:

https://brainly.com/question/12501526

#SPJ2

Can anyone Help I have 25 questions, 2hr 40mins remaining,

Answers

Combining the like-terms, the result of the addition of polynomials f(x) and g(x) is given by:

[tex]f(x) + g(x) = 21x^3 + \frac{5}{4}x^{\frac{1}{2}} + 19x^{-\frac{1}{4}} + 8x^{-4}[/tex]

How do we add polynomials?

We add polynomials combining the like-terms, that is, adding terms with the same exponent.

In this problem, the polynomials are:

[tex]f(x) = 3x^{\frac{1}{2}} + 7x^{-\frac{1}{4}} + 8x^{-4}[/tex][tex]g(x) = -\frac{7}{4}x^{\frac{1}{2}} + 12x^{-\frac{1}{4}} - 21x^3[/tex]

Combining the like terms, the addition is given by:

[tex]f(x) + g(x) = \left(3 - \frac{7}{4}\right)x^{\frac{1}{2}} + (7 + 12)x^{-\frac{1}{4}} + 8x^{-4} + 21x^3[/tex]

[tex]f(x) + g(x) = 21x^3 + \frac{5}{4}x^{\frac{1}{2}} + 19x^{-\frac{1}{4}} + 8x^{-4}[/tex]

More can be learned about addition of polynomials at https://brainly.com/question/9438778

#SPJ1

Which statements are true? Select three options O The line x = O is perpendicular to the line y = -3 O All lines that are parallel to the y-axis are vertical lines. O All lines that are perpendicular to the x-axis have a slope of O. O The equation of the line parallel to the x-axis that passes through the point (2, -6) is x = 2. O The equation of the line perpendicular to the y-axis that passes through the point (-5, 1) is y = 1

Answers

The statements which are true among the answer choices are;

The line x = O is perpendicular to the line y = -3.All lines that are parallel to the y-axis are vertical lines.

Which statements among the answer choices are true?

In the concept of straight line graphs (linear graphs) the statements are evaluated as follows;

The line x = O is perpendicular to the line y = -3. - This is true because the former is a vertical line while the latter is horizontal.

All lines that are parallel to the y-axis are vertical lines. - True, because the y-axis itself is a vertical line.

All lines that are perpendicular to the x-axis have a slope of O. - False, because all lines perpendicular to the X axis are vertical lines and have do not have slope 0.

The equation of the line parallel to the x-axis that passes through the point (2, -6) is x = 2. - False, because a line parallel to the x-axis is defined by y.

The equation of the line perpendicular to the y-axis that passes through the point (-5, 1) is y = 1. - True.

Read more on parallel and perpendicular lines;

https://brainly.com/question/7197064

#SPJ1

g(x) f(x) = −6x − 3 cosine function with y intercept at 0, negative 3 h(x) = 2 cos(x + π) − 1 Using complete sentences, explain which function has the greatest y-intercept. (10 points)

full step by step process please.

Answers

A function assigns the values. The function that has the greatest y-intercept is h(x).

What is a Function?

A function assigns the value of each element of one set to the other specific element of another set.

There are three function, g(x){given in the graph below}, f(x)= -6x-3, and -3h(x) = 2 cos(x + π) − 1, the y-intercept of the function is the point at which the graph intersects the y-axis. Therefore, the y-intercept are,

g(x) = -3

f(x) = -3

h(x) = 1

Hence, the function that has the greatest y-intercept is h(x).

Learn more about Function:

https://brainly.com/question/5245372

#SPJ1

What are the roots for the quadratic equation below?

3x^2+9x-2=0

Answers

Answer:

[tex]\sf{$a)$} \ x=\dfrac{-9\pm\sqrt{105}}{6}[/tex]

Step-by-step explanation:

Quadratic formula: [tex]x=\dfrac{-b\pm\sqrt{b^2-4ac}}{2a}[/tex]

Quadratic equation: ax² + bx + c = 0, where a ≠ 0

Given: 3x² + 9x - 2 = 0

⇒ a = 3, b = 9, c = -2

Substitute the given values into the formula and simplify:

[tex]x=\dfrac{-9\pm\sqrt{9^2-4(3)(-2)}}{2(3)}\\\\x=\dfrac{-9\pm\sqrt{81+24}}{6}\\\\x=\dfrac{-9\pm\sqrt{105}}{6}[/tex]

Therefore, the correct answer is A.

Learn more about quadratic equations here:
brainly.com/question/27638369

brainly.com/question/27740685

A 13 ​-ft ladder is leaning against a house when its base starts to slide away. By the time the base is 12 ft from the​ house, the base is moving away at the rate of 15 ​ft/sec.
a. What is the rate of change of the height of the top of the​ ladder?
b. At what rate is the area of the triangle formed by the​ ladder, wall, and ground changing​ then?
c. At what rate is the angle between the ladder and the ground changing​ then?

Answers

I think the answer is B because all the air other answers are incorrect

Work out the lengths of sides a and b.
Give your answers to 1 decimal place.
triangle a is a right angled triangle with a height | of 8cm base of 5cm and missing hypotenuse
triangle b is a right angled triangle with a height | of 12cm and a hypotenuse of 17cm with a missing base

Answers

The length of the missing sides of triangles A and B are 9.43 cm and 12.04 cm respectively.

Pythagoras theorem

Triangle A:

Height = 8cmBase = 5cmHypotenuse = x cm

Hypotenuse² = height ² + base²

x² = 8² + 5²

= 64 + 25

x² = 89

x = √89

x = 9.43 cm

Triangle B:

Height = 12cmBase = x cmHypotenuse = 17 cm

Hypotenuse² = height ² + base²

17² = 12² + x²

17² - 12² = x²

289 - 144 = x²

145 = x²

x = √145

x = 12.04 cm

Learn more about right triangle:

https://brainly.com/question/2217700

#SPJ1

Simplify the root below
\frac{a}{b} \sqrt[]{ \frac{405}{324} }

Answers

Answer:

  (√5)/2

Step-by-step explanation:

To simplify the root, we remove perfect squares.

__

Factors that are the same in numerator and denominator cancel. A square under the radical can have its root brought out of the radical.

  [tex]\sqrt{\dfrac{405}{324}}=\sqrt{\dfrac{9^2\cdot5}{9^2\cdot2^2}}=\boxed{\dfrac{\sqrt{5}}{2}}[/tex]

Other Questions
Which of the following was the main reason farmers supported the war efforts?O to show patriotismO to increase their salesO to help othersO to follow the laws= What is the answer to the question the virus known as ________ primarily infects lymphocytes. The graph below shows point P and line A. If line B is drawn such that it passes through point P and is parallel to line A, what is the equation of line B? Give your answer in the form y = mx + c, where m and care integers or fractions in their simplest forms. The graph below shows point P and line A. If line B is drawn such that it passes through point P and is parallel to line A , what is the equation of line B ? Give your answer in the form y = mx + c , where m and care integers or fractions in their simplest forms . Please help!!!What is the area of , given that =/3 radians? Express your answer in terms of and as a decimal rounded to the nearest tenth. Show all your work. Read the excerpt from chapter 8 of The Travels of Marco Polo and study the map.Let us go on now to tell of a large city which forms part of the province of Aden but has a petty ruler of its own. This city, which lies about 400 miles north-west of the port of Aden, is called Shihr. It is ruled by a count, who maintains strict justice in his domain. He has several cities and towns under his sway but is himself subject to the sultan of Aden. The people are Saracens and worship Mahomet. The city has a very good port; for I assure you that many merchant-ships come here well loaded with goods from India, and from here they export many goods to India. In particular they export innumerable fine chargers and sturdy pack-horses of great worth and price, on which the merchants make a handsome profit.Map of Europe and Asia showing Marco Polo's route in red dashes and arrows.Based on the text and the map, which body of water do the Saracens use as their port?The Mediterranean SeaThe Arabian Sea The Bay of Bengal The Spice Islands How might the design be changed so that additional copies of print statements would not be needed? when you rub neutral objects Find a rational number between -1/3 and 1/2 5. (02.01 LC)Look at the image, and then choose the option to complete the sentence with the correct informal t command. (2 points)_______ el bal del carro.A. O AbraB. Abras C. abre D. abres Getting treatment at the dentistLow riskHigh risk????????????help please A family released their pet fish family into a local pond. The fish had no known local population. The impact of the fish on the ecosystem is not yet known.Which term would best define the fish added to the pond? Tariq also designs hot tubs. The diameter of the hot tub is 3 meters. What is the area of the deck that surrounds the hot tub? If necessary, round to the nearest tenth. A plane is coming in to taxi at an airport. It lands parallel to the surface at 480m/s and takes exactly one minute to come to a complete stop. What is its acceleration? The type of transcriptional control in operons in which a regulatory protein is an activator and stimulates transcription is called. Read this excerpt from Song of the Old Mother by William Butler Yeats.I rise in the dawn, and I kneel and blowTill the seed of the fire flicker and glow;Which description gives the best visualization of this excerpt?The narrator enjoys the warmth of the fire.The narrator cooks seeds in a pot on the fire.The narrator works to cool food that has cooked on the fire.The narrator restarts the fire by blowing on the embers. A square plastic tarp has sides that are 6 meters long. what is the tarp's area What is the analine blue stained structure in the sample? (6+9)2+6(600-9)+600 divide 30 What does it mean when a solution is saturated?O The container is full.O No more solute can dissolve in solvent.O The solute has turned into a solid.O The solvent has turned into a solid.